UTNianos

Versión completa: [AM2] Calculo de la masa de un cuerpo
Actualmente estas viendo una versión simplificada de nuestro contenido. Ver la versión completa con el formato correcto.
hago otra pregunta:

Calcular la masa del cuerpo limitado por \[x^{2}+z^{2} = 4\] y \[y^{2}+z^{2} = 4\] en el primer octante, si su densidad es proporcional a la distancia desde el punto al plano xz.

Me queda un cuarto de circulo, Flax lo hace por cartesianas, pero por cilindricas no sale mas facil el circulo? yo plantee:

\[\int_{0}^{\pi /2} d\sigma \int_{0}^{2} \rho d\rho \int_{0}^{(4-\rho sen\sigma )^{1/2}} y dy\]

Pero me da 4/3k y a el 3/2k
Gara Estaria bueno si las nuevas consultas las hacen en threads nuevos, asi son mas faciles de encontrar despues para otras personas.
Dividí el tema del anterior, y cambie el título de tu mensaje por uno mas descriptivo de acorde al problema que planteas, ayudanos a mantener el foro mas organizado. thumbup3
La razón por la que en Flax se hace en cartesianas es que para este ejercicio en particular es mas sencillo hacerlo de esa manera que cambiando todo a cilindricas, porqué es un quilombo, por eso me lleva a preguntarte ¿como obtuviste los límites de integración, a que variables aplicaste el cambio de coordenadas?
(11-05-2012 20:41)Saga escribió: [ -> ]Dividí el tema del anterior, y cambie el título de tu mensaje por uno mas descriptivo de acorde al problema que planteas, ayudanos a mantener el foro mas organizado. thumbup3
La razón por la que en Flax se hace en cartesianas es que para este ejercicio en particular es mas sencillo hacerlo de esa manera que cambiando todo a cilindricas, porqué es un quilombo, por eso me lleva a preguntarte ¿como obtuviste los límites de integración, a que variables aplicaste el cambio de coordenadas?

Al ser primer octante, utilice angulos de 0 a \[\pi \]/2

El \[ \rho \] lo saque por \[x^{2} + y^{2} = 4\], por lo que te queda \[ \rho ^{2} = 4\] y despejas a \[ \rho = 0\] y \[ \rho = 2\]

Y para z despeje \[z^{2} + y^{2} = 4\] y me quedo:\[y = (4-\rho^{2} (sen\sigma)^{2} )^{1/2}\], entonces lo puse entre 0 y esa función.
Como dijo Saga, éste ejercicio sale muchísimo más fácil por cartesianas. Pero a diferencia del que te expliqué en otro topic, este no sería tan engorroso plantearlo por cilíndricas.


Planteamos las distintas regiones en cilíndricas. Para ello:

\[x=p.cos \phi \;\; \wedge \;\; z=p.sen \phi\]


Entonces:

\[x^{2}+z^{2} = 4 \to p^2=4 \to 0<p<2\]

\[y^{2}+z^{2} = 4 \to y=\sqrt{4-p^2.sen^2 \phi} \to 0<y<\sqrt{4-p^2.sen^2 \phi}\]

\[0<\sqrt{4-2^2.sen^2 \phi} \to 2^2.sen^2 \phi<4 \to sen^2 \phi < 1 \to 0<\phi< \frac{\pi}{2}\]


Por ende, la integral te queda:

\[\int_{0}^{\pi /2} d\sigma \int_{0}^{2} \rho d\rho \int_{0}^{\sqrt{4-p^2 sen^2 \phi }} y dy\]

TE OLVIDASTE EL CUADRADO DE UNO DE LOS TERMINOS DEL LIMITE SUPERIOR DE Y.

RESOLUCION_WOLFRAM=\[\frac{3\pi}{2}\]
Che maty, me parece, que hay algo mal en tu planteo, fijate que la respuesta es \[\frac{3}{2}k\] que es distinto a \[\frac{3}{2}\pi\] te esta "sobrando" el \[\pi\], ademas si analizas, si se resuelven en cartesianas, no creo que vaya a aparecer ningun \[pi\] si las trabajamos en dichas coordenadas
Esta bien lo que resolvio maty, da 3/2 k pi. (en el enunciado se me chispoteo el pi)


PD: editenlo si pueden asi ya queda bien !
Ah oks entonces no dije nada ;), grande maty thumbup3
Gracias, no fue nada igual.
Fue sólo encontrar un simple error que le pasa a cualquiera.
Saludos! thumbup3
URLs de referencia